Difference between revisions of "2017 AMC 10B Problems/Problem 6"

m (Video Solution 2)
m (Video Solution 2)
Line 13: Line 13:
 
~savannahsolver
 
~savannahsolver
  
==Video Solution 2==
+
==Video Solution by TheBeautyofMath==
 
https://youtu.be/XRfOULUmWbY
 
https://youtu.be/XRfOULUmWbY
  

Revision as of 18:48, 17 January 2021

Problem

What is the largest number of solid $2\text{-in} \times 2\text{-in} \times 1\text{-in}$ blocks that can fit in a $3\text{-in} \times 2\text{-in}\times3\text{-in}$ box?

$\textbf{(A)}\ 3\qquad\textbf{(B)}\ 4\qquad\textbf{(C)}\ 5\qquad\textbf{(D)}\ 6\qquad\textbf{(E)}\ 7$

Solution

We find that the volume of the larger block is $18$, and the volume of the smaller block is $4$. Dividing the two, we see that only a maximum of four $2$ by $2$ by $1$ blocks can fit inside the $3$ by $3$ by $2$ block. Drawing it out, we see that such a configuration is indeed possible. Therefore, the answer is $\boxed{\textbf{(B) }4}$.

Video Solution

https://youtu.be/lgdWiCz6M3c

~savannahsolver

Video Solution by TheBeautyofMath

https://youtu.be/XRfOULUmWbY

~IceMatrix

2017 AMC 10B (ProblemsAnswer KeyResources)
Preceded by
Problem 5
Followed by
Problem 7
1 2 3 4 5 6 7 8 9 10 11 12 13 14 15 16 17 18 19 20 21 22 23 24 25
All AMC 10 Problems and Solutions

The problems on this page are copyrighted by the Mathematical Association of America's American Mathematics Competitions. AMC logo.png